Information Technology Reference
In-Depth Information
Proof. Let k := dk ( M ). Then, there exists an acyclic digraph D whose double
competition multigraph is M
I k . Let ( v 1 ,...,v n + k ) be an acyclic ordering of
D .Let i be the minimum index such that v i
V ( M ) and let j be the maximum
index such that v j
j ). Since M has no isolated
vertices, the vertex v i (and v j ) has at least one incident edge. Therefore, v i (and
v j ) must have both an in-neighbor and an out-neighbor in D . Since v i has an
in-neighbor, we have i
V ( M ). Then k
( i
1)+( n + k
2. Since v j has an out-neighbor, we have j
n + k
1.
Thus we obtain k
2. Hence the lemma holds.
2 , dk ( P n )=2 .
Example 4. For a path P n of order n with n
Proof. Let P n be the path with V ( P n )=
{
v 1 ,...,v n }
and E ( P n )=
{{
v i ,v i +1 }|
i
∈{
1 ,...,n
1
}}
. We define a digraph D by V ( D )=
{
v 1 ,...,v n }∪{
a, z
}
n− 2
i =1 {
and A ( D )=
{
( a, v 1 ) , ( a, v 2 )
}∪
( v i ,v i +1 ) , ( v i ,v i +2 )
}
∪{
( v n− 1 ,v n )
}∪
{
, where a and z are new vertices. Then, the digraph D is
acyclic since the ordering ( a, v 1 ,...,v n ,z ) is an acyclic ordering of D .Wecan
easily check that the double competition multigraph of D is P n ∪{
( v n− 1 ,z ) , ( v n ,z )
}
a, z
}
.Thus
dk ( P n )
2. By Lemma 3 , dk ( P n )
2. Hence dk ( P n )=2.
Now, we give a characterization for multigraphs with bounded double multicom-
petition number, which is an extension of Theorem 1 .
Theorem 5. Let M be a multigraph with n vertices. Let k be a nonnegative
integer. Then, the double multicompetition number of M is at most k if and only
if there exist an integer t such that 0
k ,anordering ( v 1 ,...,v n ) of the
vertices of M , and a double indexed edge clique partition
t
{
S ij |
i, j
[ n + k ]
}
of
M such that the following conditions hold:
(i) For any i, j
[ n ] ,if
|
A i
B j |≥
2 , then A i
B j = S t + i,t + j ,where A i and
B j are the sets defined for i, j
[ n ] by
∪{
A i =
S t + i,t + p ,
v b−t |
v i
S ab ( a, b
∈{
t +1 ,...,t + n
}
)
}
,
p
[ n + k
t ]
∪{
B j =
S t + q,t + j ,
v a−t |
v j
S ab ( a, b
∈{
t +1 ,...,t + n
}
)
}
.
q
[ n + k
t ]
(ii) Let Λ 1 =
{
1 ,...,t
}
, Λ 2 =
{
t +1 ,...,t + n
}
,and Λ 3 =
{
t + n +1 ,...,t + n +
( k
t )(= n + k )
}
.
If ( i, j )
Λ 1 ×
Λ 2 , then S ij ↆ{
v 1 ,...,v ( j−t ) 1 }
.
If ( i, j )
Λ 1 ×
Λ 3 , then S ij ↆ{
v 1 ,...,v n }
.
If ( i, j )
Λ 2 ×
Λ 2 and i<j , then S ij ↆ{
v ( i−t )+1 ,...,v ( j−t ) 1 }
.
If ( i, j )
Λ 2 ×
Λ 3 , then S ij ↆ{
v ( i−t )+1 ,...,v n }
.
Otherwise, S ij =
.
Search WWH ::




Custom Search